(x,2) is a solution to 7x-5>y

Answers

Answer 1

No, becuase not all the posible values of x make the inequality true.

For example, if x = 0

7(0) - 5 > 2

0 - 5 > 2

- 5 > 2 This is not true.


Related Questions

How many pairs of parallel edges are there this rectangular prism

Answers

Answer:

6 i think

Step-by-step explanation:

if you count the top and bottom one, thats 2, the two sides, and the front and back. add 2+2+2 which is 6.

First person to help gets brainliest!!

Answers

The value of x from the equation represented by the given bar diagram is 20.

The equation is represented using the bar diagram and from the bar diagram be can observe that the equation will be the following equation mentioned below:

3.5 x + 12.5 x = 320                                        equation 1

The bar diagram represents 3.5 x + 12.5 x = 320  equation. Now we will simplify this equation and will find the value of x from this equation. The steps that need to be followed to simplify this equation are shown below;

3.5 x + 12.5 x = 320

16 x = 320

Divide both sides of equation by 16 , we will get

x = 320/16

x = 20

The value of x from the equation 3.5 x + 12.5 x = 320 is 20.

To know more about  bar diagram refer to the link:

https://brainly.com/question/1417364

#SPJ1

Which of the following equations describes the line shown below? Check allthat apply.(-5,6)(-1,-1)

Answers

Options E and F are the correct options

Here, we need to get the equation of the line that passes through the two points given

Mathematically, the equation of a line can be generally expressed as ;

y = mx + c

where m is the slope of the line and c is the y-intercept of the lien ( where it crosses the y-axis)

To get the slope m , we are goint to use the formula below;

[tex]\begin{gathered} m=\frac{y_2-y_1}{x_{2\text{ - }}x_1} \\ \\ x_{1\text{ }}=-5,x_2=-1,y_1\text{ = }6,y_2\text{ = -1} \end{gathered}[/tex]

Substituting these values into the slope equation , we have

m = (-1-(6))/(-1-(-5)) = -7/4

The equation is now in the form;

y = -7x/4 + c

what is left is to get the y-intercept

We can get the value of the y-intercept by substituting for x and y using any of the points

Let us use the second point

Here, x = -1 and y = -1

Thus;

-1 = -7(-1)/4 + c

-1 = 7/4 + c

c = -1-7/4 = -11/4

So the complete equation of the line is;

y = -7x/4 - 11/4

To properly answer the questions, we need the forms in the options that looks like or can be manipulated arithmetically to look like what we have as our answer

We proceed with each of the options one after the other to see if they can be written in the form of our answer

Option A

y - 5 = -7/4 ( x + 6)

Opening the bracket and evaluating;

y - 5 = -7x/4 - 42/4

y = -7x/4 -42/4 + 5

y = -7x/4 -22/4

we can see this is not a correct option

Option B

y - 6 = -4/7 (x + 5)

Multiply through by 7

7y - 42 = -4x - 20

7y = -4x - 20 + 42

7y = -4x + 22

divide through by 7

y = -4x/7 + 22/7

We can see this is incorrect also

Option C

y - 1 = -4/7(x-1)

Multiply through by 7

7y - 7 = -4x + 4

7y = -4x + 4 + 7

7y = -4x + 11

divide through by 7

y = -4x/7 + 11/7

This is incorrect too

Option D

y + 1 = -4/7 ( x + 1)

Multiply through by 7

7y + 7 = -4x - 4

7y = -4x -4 -7

7y = -4x - 11

y = -4x/7 - 11/7

This is incorrect also

Option E

y - 6 = -7/4 (x + 5)

Multiply through by 4

4y - 24 = -7x - 35

4y = -7x - 35 + 24

4y = -7x -11

y = -7x/4 - 11/4

This is a correct option

Option F

y + 1 = -7/4 ( x + 1)

Multiply through by 4

4y + 4 = -7x - 7

4y = -7x - 7-4

4y = -7x - 11

y = -7x/4 - 11/4

This is a correct option too

Which of the following is the equation that represents the graph?

Graph of a line the passes through the points negative 3 comma 0 and 0 comma negative 2.

y equals negative two thirds times x minus 3
y equals negative three halves times x minus 2
y equals negative three halves times x minus 3
y equals negative two thirds times x minus 2

Answers

y equals negative two thirds times x minus 2 is the equation of graph.

What is slope of line?

The slope of the line is the ratio of the rise to the run, or rise divided by the run.

The given two points are 3 comma 0 and 0 comma negative 2.

i.e (3,0)(0,-2)1)

The formula for slope is (y₂-y₁)/(x₂-x₁)

slope = (-2 - 0) / (0 - 3)

= -2 / -3

The negative negative are multiplied we get positive

= 2/3

The standard form of a line is y = mx + b, where m is slope of line and b is y intercept.

slope(m) = 2/3

We can use either of points (0,-2)...x = 0 and y = -2

now sub and find b, then y intercept

-2 = 2/3(0) + b

-2 = b

so  equation is : y = 2/3x - 2

Hence y equals negative two thirds times x minus 2 is the equation of graph.

To learn more on slope of line click:

https://brainly.com/question/16180119

#SPJ1

how many 4 letter code words can be made using the letters in the word pencil

Answers

Combination is a way of selecting items from a collection where the order of selection does not matter.

The number of 4-letter code words that can be made using the letters in the word pencil is 15.

What is combination?

Combination is a way of selecting items from a collection where the order of selection does not matter.

We have,

Total number of letters in the word pencil = 6

i,e P E N C I L

Combination formula:

[tex]^nC_r[/tex] = [tex]\frac{n!}{r!(n-r)!}[/tex]

Here,

n = 6

r = 4

The number of 4-letter code words that can be made using the letters in the word pencil:

[tex]^6C_4[/tex] = 6! / 4! 2! = (6 x 5) / 2= 15

Thus,

The number of 4-letter code words that can be made using the letters in the word pencil is 15.

Learn more about combination here:

brainly.com/question/2970011

#SPJ1

#8.Verify algebraically if the function is even, odd, or neither

Answers

g(x)= x^4 √(1+x)

g(-x) = (-x)^4 √1-x

g (-x) = x^4 √1-x

Since the obtained function is not the same or the opposite it is neither.

A backpack weighs 10 N. What is the mass of the backpack?

Answers

The mass of the backpack is 1.02 kilograms.

What is the mass of the backpack?

On earth, the weight of an object of mass M is given by:

W = M*g

Where g is the gravitational acceleration:

g = 9.8m/s^2

In this case, the weight is 10N, then we can write:

10N = M*9.8m/s^2

Solving this for M we get:

10N/(9.8m/s^2) = M = 1.02 kg

Learn more about weight:

https://brainly.com/question/2337612

#SPJ1

Nakeisha needs to order some new supplies for the restaurant where she works. The restaurant needs at least 566 forks. There are currently 188 forks. If each set on sale contains 8 forks, which inequality can be used to determine xx, the minimum number of sets of forks Nakeisha should buy?

Answers

The inequality that can be used to determine the minimum number of set of forks Nakeisha should buy is xx ≥ (566 - 188) / 8.

What is the minimum set of forks she should buy?

The first step is to determine the appropriate inequality signs that can be used.

> means greater than

< means less than

≥ means greater than or equal to

≤ less than or equal to

The inequality sign that would be used is ≥.

The form of the inequality is: xx ≥ (least number of forks needed - forks they current have) / number of forks in one set

xx ≥ (566 - 188) / 8

To learn more about inequality, please check: https://brainly.com/question/5031619

#SPJ1

Answer:

566 < 188 + 8x

Step-by-step explanation:

confused on this please help!!

Answers

Answer:

12.75

Step-by-step explanation:

it is the same as the other side just like the similar triangle is.

PLS HELP ASAP WILL GIVE BRAINLIST

Answers

The angles in the parallelogram is as follows:

a = 27°b = 55°c = 55°

How to find angles in a parallelogram?

A parallelogram is a quadrilateral with opposite sides equal to each other and opposite side parallel to each other.

The diagonal of a parallelogram bisect each other.

Therefore, the angles a, b and c in the parallelogram can be found as follows:

180 - 118  - 35 = a (sum of angles in a triangle)

180 - 153 = a

a = 27 degrees.

Therefore, let's find angle b.

62 + 63 + b = 180 (sum of angles in a triangle)

125 + b = 180

180 - 125 = b

b = 55 degrees

Let's find the angle c

Hence,

b = c (alternate angles)

Therefore,

c = 55 degrees.

learn more on parallelogram here: https://brainly.com/question/16578666

#SPJ1

Simplify using exponent: B³ x B⁵


HELPPPPPP

Answers

Answer:

B⁸

Step-by-step explanation:

When you have the same base, which in this case is B, you add the exponents and you get 8.

Hope this helps!

P.S, could I get Brainliest? Thanks!

I was explained this by another tutor but it didn't make too much sense

Answers

Given a right angle triangle

The hypotenuse of the triangle = x

The adjacent side to the angle 50 = 6

We will find x using the cosine function as follows:

[tex]\begin{gathered} \cos 50=\frac{adjacent}{hypotenuse} \\ \\ \cos 50=\frac{6}{x} \\ \\ x=\frac{6}{\cos 50}=9.334 \end{gathered}[/tex]

Rounding to the nearest tenth

So, the answer will be x = 9.3

Can you help me find the slope, y-intercept, and the equation based off this chart?

Answers

y-interceptis As given by the question

There are given that the value of x and y.

Now,

For finding the value of the slope, choose two-point from the given table

So,

Suppose the two points are:

[tex](1,\text{ 5) and (}3,\text{ 3)}[/tex]

Now,

From the formula of slope:

[tex]m=\frac{y_2-y_1}{x_2-x_1}[/tex]

Where,

[tex]x_1=1,y_1=5,x_2=3,y_2=3[/tex]

Put all the above value into the given formula

So,

[tex]\begin{gathered} m=\frac{y_2-y_1}{x_2-x_1} \\ m=\frac{3_{}-5_{}}{3_{}-1_{}} \\ m=-\frac{2}{2} \\ m=-1 \end{gathered}[/tex]

Hence, the value of the slope is -1.

Now,

For the equation of the line that is based on the given chart, use two supposes point and the given slope.

So,

The two poin and slope is:

[tex](1,\text{ 5) and (3, 3) and m=-1}[/tex]

Then,

From the formula of two point form;

[tex](y-y_1)=m(x-x_1)[/tex]

Where,

[tex]x_1=1,y_1=5,x_2=3,y_2=3,\text{ m=-1}[/tex]

Then,

Put all the given value into the above formula

So,

[tex]\begin{gathered} (y-y_1)=m(x-x_1) \\ (y-5_{})=-1(x-1_{}) \\ (y-5)=-x+1 \\ y-5+x-1=0 \\ y+x-6=0 \end{gathered}[/tex]

Hence, the equation of the line is:

[tex]y=-x+6[/tex]

Now,

To find the y-intercept, put x=0 and solve for x

So,

The y intercept of the given equation is:

[tex]\begin{gathered} y=-x+6 \\ y=-(0)+6 \\ y=6 \end{gathered}[/tex]

Hence, the y-intercept is (0, 6)

find 5 consecutive odd integers who's sum is -45(solve algebraically)

Answers

let the consecutive odd integers be

x, x+ 1, x+ 2, x + 3, x + 4

Average = sum / total number

Average = -45

Total number = 5

-45 = x + x + 1 + x + 2 + x + 3 + x + 4 / 5

Introduce cross multiplication

-45 x 5 = x + x + 1 + x + 2 + x + 3 + x + 4

-225 = x + x + x + x + x + 1 + 3 + 4 + 5

-225 = 5x + 16


Please help I will mark branlist

Answers

Answer: True

Step-by-step explanation:

I believe it's true since a quadratic equation can either have one real solution, two real solutions, or no real solutions. Hope this helps.

What is a(b(x)) if a(x) = 3x 2 and g(x) = -3x + 1?

Answers

The equivalent composite function is expressed as  -9x + 5

Composite functions

Composite functions are also known as function of a function.

Given the following functions:

a(x) = 3x+2 and

b(x) = -3x + 1

Determine the value of a(b(x))

a(b(x)) = a(-3x+1)

a(b(x)) = 3(-3x+1) + 2

a(b(x)) = -9x + 3 + 2

a(b(x)) = -9x + 5

This gives the equivalent composite function.

Learn more on composite function here: https://brainly.com/question/10687170

#SPJ1

Please help with this problem

Answers

Answer:

The answer is I think 16

Step-by-step explanation:

You can look at the figure beside andfind this answer

Question 1 (10 points)
Listen ▶
Twelve major Earthquakes had Richter magnitudes shown here.
6.8 6.0 7.5 6.8 6.2 6.0 7.0 5.2 6.2 6.3 7.0 5.2
Find Mean
5.35
6.35
7.35
6.95

Answers

Answer: 6.35

Step by step explanation:
To find the Mean first find the sum of all the numbers then divide by the amount of numbers you added.

6.8+6.0+7.5+6.8+6.2+6.0+7.0+5.2+6.2+6.3+7.0+5.2= 76.2

76.2/12= 6.35

Your welcome!
Hope this helps!

The graphs below have the same shape. f(x) = x2What is the equation of the graph of g(x)?10g(x)f(x)X10510-10O A. g(x) = (x+3)2B. g(x) = (x - 3)2C. g(x) = x2 - 3D. g(x) = x2 + 3

Answers

The equation of f(x) is

[tex]f(x)=x^2[/tex]

The graph of g(x) is shifted 3 units to the left. This is a horizontal translation.

For horizontal translation, we follow the following rule >>>

Let parent function be f(x), then

• f(x+a) is the parent translated "a" units left

,

• f(x-a) is the parent translated "a" units right

Thus, following the rule, we can clearly say that g(x) = f(x+3)

We can write >>

A particle moves along the x-axis so that at time t\ge 0t≥0 its velocity is given by v(t)=3t^2-18t+15.v(t)=3t2−18t+15. Determine all values of tt when the particle is at rest.

Answers

When a particle is at rest, the velocity = 0. Therefore, we equate the v(t) term to zero (0)

[tex]undefined[/tex]

If f(x)=x²-3x+5 and g(x)=-2x, find the following composition.
(f ° g)(2)
(f ° g)(2) = ____ (Simplify your answer.)

Answers

The value of the composite function (fog)(2) is equivalent to 0

Composite functions

A mathematical procedure called "function composition" combines two functions, f and g, to produce a third function, h, where g is the sum of f and

Given the functions below;

f(x) = x^2-3x+5

g(x) = -2x

Determine the composite function (gof)(2)

(fog) = f(g(x))

f(g(x)) = g(x^2-3x+5)

f(g(x)) = -2(x^2-3x+5)

f(g(2)) = -2(2^2-3(2)+2)

f(g(2)) = -2(0) = 0

This gives the equivalent composite value

Learn more on composite function here: https://brainly.com/question/10687170

#SPJ4

If a1 = 7 and an3an-1 + 4 then find the value of a4.

Answers

A1 = 7

An = 3A(n-1) + 4. According to this, A2 would be= 3*7 + 4 =25

A1 = 7, A2 = 25, then the difference is d = 18

The general formula for an aritmethic progression is An = A1 + (n -1)d

For A4 = 7 + (4-1)*18 = 61

Hey there Ms or Mr, could you please help me out with this problem? Just a head up, this my homework for today not a quiz. it's about Trapezoids.

Answers

We have to relate the angles of the trapezoid in order to find the measure of BAD.

Given:

[tex]\begin{gathered} AB\mleft\Vert DC\mright? \\ AD\cong BC \end{gathered}[/tex]

Then

[tex]\begin{gathered} \angle BAD\cong\angle ABC \\ \angle ADC\cong\angle BCD \end{gathered}[/tex]

The angles are congruent because if AD and BC are congruent, and the lines are parallel, they sides will form the same angles.

Also, as the segment AD is "cut" by two parallel lines, the angles BAD and ADC are consecutive interior angles and are, therefore, supplementary.

Then, we can write:

[tex]\begin{gathered} m\angle BAD+m\angle ADC=180\degree\text{ and }\angle ADC=\angle BCD \\ \Rightarrow m\angle BAD+m\angle BCD=180\degree \\ 2x+(3x-5)=180 \\ 5x-5=180 \\ 5x=180+5 \\ 5x=185 \\ x=\frac{185}{5} \\ x=37 \end{gathered}[/tex]

Now, with the value of x, we can calculate the measure of BAD as:

[tex]m\angle BAD=2x=2\cdot37=74\degree[/tex]

Answer: the measure of BAD is 74°.

I'll give brainliest!!

Answers

1. 1/y squared
2. Y of the power of 4
3.Y of the power of 9
4.Y of the power of 6

I want answer. I need t’s solution.And I want explanation too.

Answers

we have the equation

[tex]log(\frac{1,000}{253.95})=12tlog(1+\frac{0.08}{12})[/tex]

Solve for t

That means

Isolate the variable t

[tex]t=\frac{log(\frac{1,000}{253.95})}{12log(1+\frac{0.08}{12})}[/tex][tex]t=17.2[/tex]

Solve for m.m- 15 = 20

Answers

Answer:

m=35

Explanation:

Given the equation:

[tex]m-15=20[/tex]

We solve for the value of m by adding 15 to both sides.

[tex]\begin{gathered} m-15+15=20+15 \\ m+0=35 \\ m=35 \end{gathered}[/tex]

The value of m is 35.

First add 15 from both sides.
The result will be m=35

Evaluate f(3) for f(x)=2x+6

Answers

[tex]\begin{gathered} f(3)=2\times3+6 \\ f(3)=6+6 \\ f(3)=12 \end{gathered}[/tex]

Suppose that Kevin's checking account shows a balance of $910.25 at the beginning of the month. During this month, he makes deposits of $175.35, $472.25, $235.40, and $450.15, and he writes checks for $307.50, $289.25, $150.30, and $327.00. Find the end-of-the-month balance in his account.

Answers

Answer:

The answer is $1,169.35

Step-by-step explanation:

Deposits is when you put money into your account, so, you'll add what he deposits. which will be $2,242.4, Next when he writes a check it means he took money out of his account so you'll subtract. You'll add up the amount of checks he wrote and subtract your answer with the deposits so, $2,243.4, - $1,074.05 = 1,169.35 and thats how you get your answer.

negative 37 over 24 times j minus 17 over 12

negative 37 over 24 times j plus 17 over 12

43 over 24 times j plus 1 over 12

negative 43 over 24 times j plus negative 1 over 12

Simplify the expression.

the expression negative one eighth j plus two thirds minus the expression five thirds j plus nine twelfths

negative 37 over 24 times j minus 17 over 12
negative 37 over 24 times j plus 17 over 12
43 over 24 times j plus 1 over 12
negative 43 over 24 times j plus negative 1 over 12


Which expression is equivalent to 3(−4.5b − 2.1) − (6b + 0.6)?

19.5b + 1.5
−19.5b − 1.5
−19.5b − 6.9
19.5b − 6.9

Answers

The Simplification of the expression negative one eighth j plus two thirds minus the expression five thirds j plus nine twelfths is "negative 43 over 24 times j plus negative 1 over 12". option D

The expression which is equivalent to 3(−4.5b − 2.1) − (6b + 0.6) is −19.5b − 6.9. option c

Simplification of fraction

negative one eighth j plus two thirds minus the expression five thirds j plus nine twelfths

= -1/8j + 2/3 - (5/3j + 9/12)

= -1/8j + 2/3 - 5/3j - 9/12

= -1/8j - 5/3j + (2/3 - 9/12)

= (-3j-40j / 24 + (8-9) / 12

= -43/24j + (-1)/12

The expression equivalent to 3(−4.5b − 2.1) − (6b + 0.6)

open parenthesis

= -13.5b - 6.3 - 6b - 0.6

collect like terms

= -13.5b - 6b - 6.3 - 0.6

= - 19.5b - 6.9

So therefore, the equivalent expression to 3(−4.5b − 2.1) − (6b + 0.6) is - 19.5b - 6.9

Read more on fraction:

https://brainly.com/question/11562149

#SPJ1

Central Bank pays 8.5% interest, compounded daily, from the date of deposit to the date of withdrawal. The daily interest rate (to the nearest millionths place) forJuly 28 (92 days in the quarter) on this account is:0.000198.0.0002070.000231.0.000216None of these choices are correct.

Answers

ofGiven the yearly interest rate as 8.5%

To find the daily interest, we will divide the yearly interest rate by 365 as shown below:

[tex]\text{daily interest rate=}\frac{yearly\text{ interest rate}}{365}[/tex]

[tex]\text{daily interest rate=}\frac{\frac{8.5}{100}}{365}=\frac{0.085}{365}=0.0002328767[/tex]

Hence, the daily interest rate (to the nearest millionths place)= 0.000232, The third option

Other Questions
What is the slope of the line that goes through the points (1,-5) and (4,1)?OA-4/3OB.-3/4. 1/2OD. 2 LESSON Sine and Cosine Ratios 13-2 Practice and Problem Solving: A/B After verifying that the triangle to the right is a right triangle, use a calculator to find the given measures. Give ratios to the nearest hundredth and angles to the nearest degree. 1. Use the Pythagorean Theorem to confirm that the triangle is a right triangle. Show your work. You have maxed out your $800 credit card from Utopian One Bank. The interest on the card is 21%. Find the interest and the final cost you pay on this bill. (Hint this problem is computed like sales tax) what is an equation of the line that passes through the points (-4, 5) and (-4, -2) PLEASEEEE HELPPP!!!! once winston saw three men who had been arrested, confessed, and reinstated in the party. a little while later they were arrested. they confessed again and were killed. what did winston realize from this experience? How do you think Sojourner Truth reacted to the 15th amendment that only gave the right to vote to men? can you help me i have to see if it is a direct variation 1. Squares with side lengths 6, 8, and 10 meters?2. Squares with areas 64 in?, 100 in?, 144 in2? 3. Two squares with side length 5 feet and a square with area 50 square feet?4. Explain how you know whether three squares will join at their corners to form a right triangle. Suppose that the annual rate of return for a common biotechnology stock is normally distributed with a mean of 5% and a standard deviation of 6%. Find the probability that the one-year return of this stock will be negative. Round to four decimal places. why do surface waves compared to body waves have a high amplitude? [tex] \rm\int_{0}^{ \frac{\pi}{2} } \frac{1}{ \sqrt{1 - {sin}^{2} ( \frac{1}2) {sin}^{2} \varphi } } d \varphi \\ [/tex] One of the drawbacks of the place theory is that it fails to adequately explain pitch perception of __________ frequency sounds. a. low b. mid-range c. high d. low and mid-range what did the french revolution accomplish? how much has endured to the current day, and what has not endured? What are the sum of all the factors of 50 which are between 1 and 50. A. 42B. 2550C. 92D. 15 if jack refinished his basement himself, the value of his work group of answer choices is included in gross domestic product (gdp). is included in the next year's gross domestic product (gdp). is only included in gross domestic product (gdp) if he uses u.s. made components. is not included in gross domestic product (gdp). how high a hill can a car coast up (engine disengaged) if friction is negligible and its initial speed is 24.4 m/s? m (b) if, in actuality, a 750 kg car with the same initial speed is observed to coast up a hill to a height 14.3 m above its starting point, how much thermal energy was generated by friction? a monopolistically competitive company assembles and installs solar panels it imports at $10 per unit. all remaining costs of the company, denoted rc, is given by the following (as a function of per unit solar panel): Change 36 inches into feet given the fact that 1 foot = 12 inches. Identify the exclusive powers given to the House of Representatives